0 Daumen
593 Aufrufe

Komme immer auf das falsche Ergebnis:

\( \frac{m}{m+1}+\frac{2 m}{m^{2}-1}+m \)

Der Nenner im zweiten Glied ist die 3. binomische Formel, und das erste Glied muss mit ( m - 1 ) erweitert werden.

\( \frac{m(m-1)+2 m}{(m+1)(m-1)}+m \)

Jetzt muss ich das dritte Glied, also + m noch erweitern.

Wenn ich ihn mit ( m² - 1 ) erweiter habe ich ja eine kubische Zahl und weiss nicht wie ich diese weg bekomme.

Nun denke ich mir man erweitert mit ( m + 1 ) ( m - 1 ).

\( \frac{m^{2}-1 m+2 m+m(m+1)(m-1)}{(m+1)(m-1)} \)

Ab hier weiss ich nicht weiter ( falls überhaupt richtig ) wie bzw. was ich kürzen soll.

Avatar von

3 Antworten

0 Daumen
 
Beste Antwort
m/(m+1) + 2m/(m^2-1) + m | Nenner überall auf m^2+1 bringen

m(m-1)/(m^2-1) + 2m/(m^2-1) + m(m^2-1)/(m^2-1) =

(m^2 - m + 2m + m^3 - m)/(m^2 - 1) =

(m^3 + m^2)/(m^2 - 1) =

(m^3 + m^2)/[(m + 1)(m - 1)]

Jetzt kürzen durch m + 1; zunächst Polynomdivision:

(m^3 + m^2) : (m + 1) = m^2

Endergebnis:

m^2/(m - 1)
Avatar von 32 k
+1 Daumen

Sieht ja soweit ganz gut aus .

Den Zähler erst einmal zusammenfassen

m²+m+m( (m+1)*(m-1)= m(m+1)+m(m+1)*,(m-1)    |nun das Distributivgesetz anwenden

                                       m(m+1)*(1+(m-1)  )         

                                     = m²(m+1)     

nun mit dem Nenner Kürzen

m²(m+1)

-------------          =m²/(m-1)

(m+1)*(m-1)

Avatar von 40 k
0 Daumen
Was sollte denn da rauskommen?

Ich habe hier eine Lösung, aber erst würde ich gern das Ergebnis erfahren, damit ich das überprüfen kann.
Avatar von

Ein anderes Problem?

Stell deine Frage

Ähnliche Fragen

Willkommen bei der Mathelounge! Stell deine Frage einfach und kostenlos

x
Made by a lovely community